LSAT and Law School Admissions Forum

Get expert LSAT preparation and law school admissions advice from PowerScore Test Preparation.

 eober
  • Posts: 107
  • Joined: Jul 24, 2014
|
#16351
Hi,

I couldn't be sure whether A or E was the correct answer choice because they both seem to weaken the argument. Could you explain why we eliminated A? Is it because we don't know about the most extensive division of labor?

Thanks!
 Robert Carroll
PowerScore Staff
  • PowerScore Staff
  • Posts: 1787
  • Joined: Dec 06, 2013
|
#16363
eober,

Answer choice (A) does not attack the conclusion that lenders should avoid loans to worker-owned businesses. Because the answer choice merely shows that some businesses with the most extensive divisions of labor have some failing, and we already know that worker-owned businesses are not the ones with the most extensive divisions of labor, this answer choice is merely showing that perhaps there is some reason for lenders to avoid even some investor-owned businesses. But even if lenders did that, it wouldn't necessarily mean that they WOULD lend money to worker-owned businesses - perhaps lenders would be well-advised not to lender money to businesses at all, if they're all risky!

We need to show that worker-owned businesses are not as risky as the stimulus says, not that NON-worker-owned businesses are more risky, as that's not directly relevant to the conclusion.

It's also true that the phrase "most extensive" only refers to some businesses, so this wouldn't tell anyone even to avoid investor-owned businesses, because some of them might not be risky or have this flaw mentioned in answer choice (A). Whatever the phrase refers to, it doesn't show that worker-owned businesses are less risky, as we want.

Robert Carroll
User avatar
 LSATStudent2023
  • Posts: 5
  • Joined: Mar 29, 2022
|
#96205
Hey Powerscore Team,

Could you explain how to weaken this argument with the answer choices given? I'm having trouble understanding why E is the correct answer or how it weakens the argument. Thanks!
User avatar
 katehos
PowerScore Staff
  • PowerScore Staff
  • Posts: 184
  • Joined: Mar 31, 2022
|
#96239
Hi LSATStudent2023!

In the stimulus, we see that Perry argues lenders should not make loans to worker-owned businesses because their inefficiencies increase the risk for lenders. So, the best way to weaken the argument is to show some way in which worker-owned businesses are NOT as risky as Perry claims them to be. Our prephrase should be something along these lines and any example of something making worker-owned businesses more efficient (and thus less risky) will do the trick.

Answer choice (E) does just that! If most worker-owned businesses compensate for inefficiencies by worker longer hours than those in investor-owned businesses, this makes worker-owned businesses less risky than Perry claims.

I hope this helps :)
Kate

Get the most out of your LSAT Prep Plus subscription.

Analyze and track your performance with our Testing and Analytics Package.